Asthma Fitz Flashcards

1
Q
  1. Which of the following best describes asthma?
    A. intermittent airway inflammation with occasional
    bronchospasm
    B. a disease of bronchospasm that leads to airway
    inflammation
    C. chronic airway inflammation with superimposed
    bronchospasm
    D. relatively fixed airway constriction
A

C. chronic airway inflammation with superimposed

bronchospasm

How well did you know this?
1
Not at all
2
3
4
5
Perfectly
2
Q
  1. The patient you are evaluating is having a severe asthma flare. You have assessed that his condition is appropriate for office treatment. You expect to find the following on physical examination:
    A. tripod posture.
    B. inspiratory crackles.
    C. increased vocal fremitus.
    D. hyperresonance on thoracic percussion.
A

D. hyperresonance on thoracic percussion.

How well did you know this?
1
Not at all
2
3
4
5
Perfectly
3
Q
  1. A 44-year-old man has a long-standing history of
    moderate persistent asthma that is normally well
    controlled by fluticasone with salmeterol (Advair®)
    via metered-dose inhaler, one puff twice a day, and
    the use of albuterol 1 to 2 times a week as needed for
    wheezing. Three days ago, he developed a sore throat, clear nasal discharge, body aches, and a dry cough. In the past 24 hours, he has had intermittent
    wheezing that necessitated the use of albuterol,
    two puffs every 3 hours, which produced partial
    relief. Your next most appropriate action is to
    obtain a:
    A. chest radiograph.
    B. measurement of oxygen saturation (SaO2).
    C. spirometry measurement.
    D. sputum smear for white blood cells (WBCs).
A

C. spirometry measurement.

How well did you know this?
1
Not at all
2
3
4
5
Perfectly
4
Q
  1. You examine Jane, a 24-year-old woman who has an acute asthma flare following a 3-day history of upper respiratory tract symptoms (clear nasal discharge, dry cough, no
    fever) . She has a history of moderate persistent asthma that is in good control and an acceptable peak expiratory flow (PEF). She is using budesonide (Pulmicort®) and albuterol as directed and continues to have difficulty with coughing and wheezing. At home, her PEF is 55% of personal best. In the office, her forced expiratory volume at 1 second (FEV1) is 65% of predicted. Her medication regimen should be adjusted to include:

A. theophylline.
B. salmeterol (Serevent®).
C. prednisone.
D. montelukast (Singulair®).

A

C. prednisone.

How well did you know this?
1
Not at all
2
3
4
5
Perfectly
5
Q
5. For Jane in question 4, you also prescribe:
A. amoxicillin.
B. azithromycin.
C. levofloxacin.
D. no antimicrobial therapy.
A

D. no antimicrobial therapy.

How well did you know this?
1
Not at all
2
3
4
5
Perfectly
6
Q
  1. Peak expiratory flow meters:

A. should only be used in the presence of a medical
professional.
B. provide a convenient method to check lung function at home.
C. are as accurate as spirometry.
D. should not be used more than once daily.

A

B. provide a convenient method to check lung function at home.

How well did you know this?
1
Not at all
2
3
4
5
Perfectly
7
Q
  1. Which of the following is most likely to appear on a
    chest radiograph of a person during an acute severe
    asthma attack?
    A. hyperinflation
    B. atelectasis
    C. consolidation
    D. Kerley B signs
A

A. hyperinflation

How well did you know this?
1
Not at all
2
3
4
5
Perfectly
8
Q
8. A 36-year-old man with asthma also needs antihypertensive therapy. Which of the following products should you avoid prescribing?
A. hydrochlorothiazide
B. propranolol
C. amlodipine
D. enalapril
A

B. propranolol

How well did you know this?
1
Not at all
2
3
4
5
Perfectly
9
Q
  1. Which of the following is inconsistent with the presentation of asthma that is not well controlled?
    A. a troublesome nocturnal cough at least 2 nights
    per week
    B. need for albuterol to relieve shortness of breath
    at least twice a week
    C. morning sputum production
    D. two or more exacerbations/year requiring oral
    corticosteroids
A

C. morning sputum production

How well did you know this?
1
Not at all
2
3
4
5
Perfectly
10
Q
  1. The cornerstone of moderate persistent asthma drug therapy is the use of:

A. oral theophylline.
B. mast cell stabilizers.
C. short-acting beta2-agonists (SABA).
D. inhaled corticosteroids.

A

D. inhaled corticosteroids.

How well did you know this?
1
Not at all
2
3
4
5
Perfectly
11
Q
  1. Sharon is a 29-year-old woman with moderate
    persistent asthma. She is not using prescribed inhaled
    corticosteroids, but is using albuterol PRN to relieve
    her cough and wheeze with reported satisfactory
    clinical effect. Currently she uses about two albuterol
    metered-dose inhalers per month and is requesting
    a prescription refill. You consider that:

A. her asthma is well controlled and albuterol use can
continue.
B. excessive albuterol use is a risk factor for asthma
death.
C. her asthma is not well controlled and salmeterol
(Serevent®) should be added to relieve bronchospasm
and reduce her albuterol use.
D. her asthma has better control with albuterol than
inhaled corticosteroids.

A

B. excessive albuterol use is a risk factor for asthma

death.

How well did you know this?
1
Not at all
2
3
4
5
Perfectly
12
Q
  1. In the treatment of asthma, leukotriene receptor
    antagonists should be used as:
    A. controllers to prevent bronchospasm.
    B. controllers to inhibit inflammatory responses.
    C. relievers to treat acute bronchospasm.
    D. relievers to treat bronchospasm and inflammation.
A

B. controllers to inhibit inflammatory responses.

How well did you know this?
1
Not at all
2
3
4
5
Perfectly
13
Q
  1. According to the National Asthma Education and
    Prevention Program Expert Panel Report 3 (NAEPP
    EPR-3) guidelines, which of the following is not a
    risk for asthma death?
    A. hospitalization or an emergency department visit for
    asthma in the past month
    B. current use of systemic corticosteroids or recent
    withdrawal from systemic corticosteroids
    C. difficulty perceiving airflow obstruction or its
    severity
    D. rural residence
A

D. rural residence

How well did you know this?
1
Not at all
2
3
4
5
Perfectly
14
Q
  1. An 18-year-old high school senior presents, asking for a letter stating that he should not participate in gym class because he has asthma. The most appropriate action is to:
    A. write the note because gym class participation could trigger asthma symptoms.
    B. excuse him from outdoor activities only to avoid
    pollen exposure.
    C. assess his level of asthma control and make changes in his treatment plan if needed so he can participate.
    D. write a note excusing him from gym until his
    follow-up examination in 2 months.
A

C. assess his level of asthma control and make changes in his treatment plan if needed so he can participate.

How well did you know this?
1
Not at all
2
3
4
5
Perfectly
15
Q
  1. You see a 34-year-old man with moderate persistent asthma who has an asthma flare and a regimen of oral prednisone is being considered. Which of the following is true?
    A. A taper is needed for prednisone therapy lasting
    longer than 4 days.
    B. A taper is not needed if the prednisone regimen is
    for 7 days or less.
    C. A taper is not needed regardless of duration of
    prednisone therapy.
    D. A taper is needed if the patient is taking concomitant inhaled corticosteroids.
A

B. A taper is not needed if the prednisone regimen is

for 7 days or less.

How well did you know this?
1
Not at all
2
3
4
5
Perfectly
16
Q
16. After inhaled corticosteroid is initiated, improvement in control is usually seen:
A. on the first day of use.
B. within 2 to 8 days.
C. in about 3 to 4 weeks.
D. in about 1 to 2 months.
A

B. within 2 to 8 days.

17
Q
  1. Compared with albuterol, levalbuterol (Xopenex®) has:
    A. a different mechanism of action.
    B. the ability potentially to provide greater bronchodilation with a lower dose.
    C. an anti-inflammatory effect similar to that of an
    inhaled corticosteroid.
    D. a contraindication to use in elderly patients.
A

B. the ability potentially to provide greater bronchodilation with a lower dose.

18
Q
  1. Which of the following is consistent with the NAEPP
    comment on the use of inhaled corticosteroids (ICS)
    for a child with asthma?
    A. The potential but small risk of delayed growth with
    ICS is well balanced by their effectiveness.
    B. ICS should be used only if a leukotriene receptor
    antagonist fails to control asthma.
    C. Permanent growth stunting is consistently noted
    in children using ICS.
    D. A leukotriene receptor antagonist are equal in
    therapeutic effect to the use of a long-acting
    beta2-agonist.
A

A. The potential but small risk of delayed growth with

ICS is well balanced by their effectiveness.

19
Q
19. A potential adverse effect from ICS use is:
A. oral candidiasis.
B. tachycardia.
C. gastrointestinal upset.
D. insomnia.
A

A. oral candidiasis.

20
Q
  1. Clinical findings characteristic of poorly controlled
    asthma include all of the following except:
    A. a recurrent spasmodic cough that is worse
    at night.
    B. recurrent shortness of breath and chest tightness
    with exercise.
    C. a congested cough that is worse during the day.
    D. wheezing with and without associated respiratory
    infections.
A

C. a congested cough that is worse during the day.

21
Q
21. Which of the following best describes the mechanism of action of short-acting beta2-agonists?
A. reducer of inflammation
B. inhibition of secretions
C. modification of leukotrienes
D. smooth muscle relaxation
A

D. smooth muscle relaxation

22
Q
  1. Regarding the use of long-acting beta2-agonists
    (LABAs), which of the following is true?
    A. LABAs enhance the ant-iinflammatory action of
    corticosteroids.
    B. Use of LABAs is associated with a small increase in
    risk of asthma death.
    C. LABA use reduces the risk of asthma
    exacerbations.
    D. LABAs can be used as monotherapy to relieve
    bronchospasms in asthma.
A

B. Use of LABAs is associated with a small increase in

risk of asthma death.

23
Q
  1. Which of the following is the therapeutic objective of using inhaled ipratropium bromide?
    A. as an anti-inflammatory.
    B. an increase in vagal tone in the airway
    C. inhibition of muscarinic cholinergic receptors
    D. an increase in salivary and mucous secretions
A

C. inhibition of muscarinic cholinergic receptors

24
Q
  1. Which of the following is true regarding the use
    of systemic corticosteroids in the treatment of
    asthma?
    A. Frequent short bursts are preferred over daily
    inhaled corticosteroids.
    B. The oral corticosteroid should be started at day 3–4
    of the asthma flare for optimal effect.
    C. The oral route is preferred over parenteral
    therapy.
    D. The adult dose to treat an asthma flare should
    not exceed the equivalent of prednisone 40 mg
    daily.
A

C. The oral route is preferred over parenteral

therapy.

25
Q
  1. Compared with short-acting beta2-agonists, long-acting beta2-agonists:
    A. are recommended as a first-line therapy in mild
    intermittent asthma.
    B. have a significantly different pharmacodynamic
    profile.
    C. have a rapid onset of action across the drug
    class.
    D. should be added to therapy only when ICS
    use does not provide adequate asthma
    control.
A

D. should be added to therapy only when ICS
use does not provide adequate asthma
control.

26
Q
  1. Which of the following statements is false regarding the use of omalizumab (Xolair®)?
    A. Its use is recommended for patients with mild
    persistent asthma to prevent asthma flares.
    B. The medication selectively binds to immunoglobulin
    E (IgE) to reduce exacerbations.
    C. Labeled indication is for patients with poorly
    controlled asthma with frequent exacerbations.
    D. Special evaluation is required prior to its use and
    ongoing monitoring is needed during use.
A

A. Its use is recommended for patients with mild

persistent asthma to prevent asthma flares.

27
Q
  1. Subcutaneous immunotherapy is recommended for
    use in patients:
    A. with well-controlled asthma and infrequent
    exacerbations.
    B. with allergic-based asthma.
    C. with moderate persistent asthma who are intolerant
    of ICS.
    D. with poorly-controlled asthma who fail therapy with
    omalizumab.
A

B. with allergic-based asthma.

28
Q
  1. Most prescribers are well-versed in the relative
    potency of ICS and prescribe an appropriate dose
    for the patient’s clinical presentation.

True or False

A

False

29
Q
  1. Approximately 80% of the dose of an ICS is
    systemically absorbed.

True or False

A

False

30
Q
  1. A leukotriene modifier and an ICS are interchangeable clinically because both groups of
    medications have equivalent anti-inflammatory
    effect.

True or False

A

False

31
Q
  1. Little systemic absorption of mast cell stabilizers
    occurs with inhaled or intranasal use.

True or False

A

True

32
Q

Due to safety concerns, mast cell stabilizers are
no longer available.

True or False

A

False

33
Q

Is spirometry or peak flow meter needed to make a diagnosis of asthma?

A

Spirometry is needed to make the diagnosis of asthma.